LSAT and Law School Admissions Forum

Get expert LSAT preparation and law school admissions advice from PowerScore Test Preparation.

 Administrator
PowerScore Staff
  • PowerScore Staff
  • Posts: 8916
  • Joined: Feb 02, 2011
|
#23565
Complete Question Explanation

Must be True. The correct answer choice is (C)

The curator in the stimulus has provided evidence supporting a conclusion that the cloak in Veronese's painting was not originally painted red by the artist, and therefore the proposed restoration to the green underneath is justified. The critic, in response, has provided evidence that the cloak was likely red at the time of Veronese's death. To get to a prephrased answer here, we must combine our knowledge that the proposed restoration will change the cloak from red to green, with the art critic's argument that Veronese originally painted to cloak red.

Answer Choice (A): While this answer may be true, the critic's statement does not provide strong support for it. All we know is that a copy was made and the cloak was red. We have no information about the quality of the copy or whether it would be distinguishable from the original.

Answer Choice (B): The curator's claim relies on technology, not the critic's. Since the critic provides no information, insight, or opinion regarding the use of technology, this cannot be the correct answer choice.

Answer Choice (C): This is the correct answer choice. The information provided by the art critic tells us that the cloak was likely red at the time of Veronese's death. Therefore, if the proposed restoration is going to change the cloak from red to green, it follows that the restoration would not restore the painting to the appearance it had at the end of Veronese's lifetime.

Answer Choice (D): The argument made by the art critic provides no information sufficient to make a value judgment on whether or not an artist's work is compromised when that work is tampered with by later artists.

Answer Choice (E): This one is tricky because it seems to match with what the art critic is implying. However, if you look at the art critic's statement more closely, you will see that she is solely making a statement on what color the cloak was at the time of Veronese's death. There is no information that can be derived from the art critic's statement regarding Veronese's original intent.
 jessamynlockard
  • Posts: 42
  • Joined: Jan 15, 2018
|
#45170
I know this is picayune, but I got throw by the stimulus saying "so soon after Veronese's death" and answer choice C saying "at the end of the artist's lifetime". Isn't "the end of the artist's lifetime" before Veronese's death?

If that's a correct interpretation, was I supposed to infer from the critic's statement that the cloak was likely modified before his death?

Thanks!
Jessamyn
 Malila Robinson
PowerScore Staff
  • PowerScore Staff
  • Posts: 296
  • Joined: Feb 01, 2018
|
#45256
Hi Jessamyn,
The end of a person's life is when the person dies. So this would mean Veronese died (his life ended) and then the painting was changed.
Hope that helps,
Malila
 FutureLawStudent
  • Posts: 2
  • Joined: May 21, 2018
|
#47777
Hi,

I had a quick question about the prephrase analysis. In this analysis it says that "Veronese originally painted the cloak red,"and when I initially read the stimulus I thought that was too far of a leap. I interpreted the information in the art critic's argument that the cloak was red shortly after Veronese's death then the cloak color was probably red and had been changed before the artist's death. Since the critic adds that the change was unlikely done by the copyist so soon after the artist's death how does that provide enough support to conclude that Veronese made the change? Just because there is enough support for the inference that the cloak color changed before Veronese death how do we know that it wasn't someone else who made the change?

Thanks!
 Adam Tyson
PowerScore Staff
  • PowerScore Staff
  • Posts: 5153
  • Joined: Apr 14, 2011
|
#47798
We don't know that, FutureLawStudent! You're correct that it's a leap too far to claim that. But the correct answer, answer choice C, doesn't require us to know who painted it red, whether Veronese or someone tampering with it after he was done. All we can get from the stimulus is that the cloak was probably red at the time Veronese died. Maybe Veronese changed it, maybe someone else, but if the art critic is right about a copyist being unlikely to make a change like that so soon after his death, the change from green to red seems likely to have occurred before he died. In other words, the copy would have been faithful to what the painting looked like at the time of his death, regardless of what it looked like originally or who changed the original or when they did it.
 portilloa3
  • Posts: 5
  • Joined: Dec 21, 2018
|
#61445
Question about the stem:

The question stem reads as follows:
"The art critic's response to the curator would provide the strongest support for which one of the following conclusions?"

Powerscore categorizes this stem as a Must Be True stem.

My confusion is how this is not categorized as a Most Strongly Supported stem.

What is the reasoning for categorizing this as a Must Be True stem instead of a Most Strongly Supported stem?

Thanks!
 Adam Tyson
PowerScore Staff
  • PowerScore Staff
  • Posts: 5153
  • Joined: Apr 14, 2011
|
#61447
In our parlance, portilloa3, a Most Strongly Supported question (which this is) is a subset of a Must Be True. We approach them in the same way, using the Fact Test, rather than treating MSS as a distinct question type. MSS questions do have a lower standard of proof than MBT, in that the correct answer need not be proven by the stimulus the way a MBT is, but since we still base our answer solely on what we read, rejecting outside information, and since the flow of information is from the stimulus to the answer choices, with the stimulus supporting the correct answer, we just put them in the same category. Good eye, though, catching the distinction! It's like the difference between Justify the Conclusion and Strengthen - one proves, the other helps, and the one that helps COULD prove but doesn't have to. A MBT stimulus proves the correct answer, while a MSS stimulus helps the correct answer (and COULD prove it).
User avatar
 a.hopp
  • Posts: 15
  • Joined: May 15, 2023
|
#101845
Could you go into more detail about why E is incorrect?

I read the Art critic's response as replying to the Curator's main conclusion, which I understood to be the first sentence of their argument: "The decision to restore the cloak...is fully justified."

It seemed the conclusion most strongly supported by the art critics response could be a sort of reply to the Curator's conclusion.

The curator believes the restoration is justified. The Art critic says it is likely the cloak was red all along. Therefore, Veronese did not intend for the cloak to be green. The restoration would not be justified.

I know this question is not asking us to find the conclusion that undermines the Curator's. I just need a bit more explanation on E's insufficiency.

Thank you!
User avatar
 Jeff Wren
PowerScore Staff
  • PowerScore Staff
  • Posts: 389
  • Joined: Oct 19, 2022
|
#101851
Hi a.hopp,

The key to ruling out Answer E is the word "originally."

We do not know what color Veronese originally intended to paint the cloak. If anything, the fact that the cloak was painted green before later being painted red (since the green paint was found under the red paint) suggests that Veronese probably did originally intend to paint the cloak green and later changed his mind (or alternatively, someone else later painted it red.)

Get the most out of your LSAT Prep Plus subscription.

Analyze and track your performance with our Testing and Analytics Package.